Paste
Copy
Cut
Options
  • Pregunta: Un experimentador ha preparado un nivel de dosis de fármaco que, según ella, inducirá sueño para al menos el 80% de las personas que sufren de insomnio. Después de examinar la dosis, creemos que sus afirmaciones con respecto a la eficacia de la dosis están infladas. En un intento de refutar su afirmación, le administramos la dosis prescrita a veinte

    Un experimentador ha preparado un nivel de dosis de fármaco que, según ella, inducirá
    sueño para al menos el 80% de las personas que sufren de insomnio. Después de examinar
    la dosis, creemos que sus afirmaciones con respecto a la eficacia de la
    dosis están infladas. En un intento de refutar su afirmación, le administramos la dosis prescrita
    a veinte insomnes, y observamos Y, el número para el cual la dosis de la droga
    induce el sueño. Deseamos contrastar la hipótesis Ho p = 0,8 frente a la alternativa Ha p < 0,8
    con región de rechazo Y <= 12

    a) En términos de este problema, ¿qué es un error tipo I?

    b) Hallar alfa.

    c) En términos de este problema, ¿qué es un error tipo II?

    d) Encuentre beta cuando p = 0.6.

    e) beta cuando p = 0.4

    f) Encuentra la región de rechazo de la forma {y mayor o igual a c} tal que α ≈ .01

    g) Para la región de rechazo en la parte (a) encuentre β cuando p = .6

    h) Para la región de rechazo en la parte (a) encuentre β cuando p = .4

  • Chegg Logo
    Esta pregunta aún no se resolvió!
    ¿No es lo que buscas?
    Envía tu pregunta a un experto en la materia.